Difference between revisions of "2002 AMC 10B Problems/Problem 20"

(Easiest Solution)
m (Solution 2: added WLOG addendum)
(6 intermediate revisions by 6 users not shown)
Line 1: Line 1:
== Problem ==
+
==Problem==
 
 
 
Let a, b, and c be real numbers such that <math>a-7b+8c=4</math> and <math>8a+4b-c=7</math>. Then <math>a^2-b^2+c^2</math> is  
 
Let a, b, and c be real numbers such that <math>a-7b+8c=4</math> and <math>8a+4b-c=7</math>. Then <math>a^2-b^2+c^2</math> is  
  
 
<math> \mathrm{(A)\ }0\qquad\mathrm{(B)\ }1\qquad\mathrm{(C)\ }4\qquad\mathrm{(D)\ }7\qquad\mathrm{(E)\ }8 </math>
 
<math> \mathrm{(A)\ }0\qquad\mathrm{(B)\ }1\qquad\mathrm{(C)\ }4\qquad\mathrm{(D)\ }7\qquad\mathrm{(E)\ }8 </math>
  
== Solution ==
+
==Solution==
 +
===Solution 1===
  
 
Rearranging, we get <math>a+8c=7b+4</math> and <math>8a-c=7-4b</math>
 
Rearranging, we get <math>a+8c=7b+4</math> and <math>8a-c=7-4b</math>
Line 13: Line 13:
 
Adding the two equations and dividing by <math>65</math> gives <math>a^2+c^2=b^2+1</math>, so <math>a^2-b^2+c^2=\boxed{(\text{B})1}</math>.
 
Adding the two equations and dividing by <math>65</math> gives <math>a^2+c^2=b^2+1</math>, so <math>a^2-b^2+c^2=\boxed{(\text{B})1}</math>.
  
== Easiest Solution ==
+
===Solution 2===
 
 
The easiest way is to assume a value for <math>a</math> and then solving the system of equations. For <math>a = 1</math>, we get the equations
 
  
 +
The easiest way is to assume a value for <math>a</math> and then solve the system of equations. For <math>a = 1</math>, we get the equations
 
<math>-7b + 8c = 3</math> and
 
<math>-7b + 8c = 3</math> and
 
 
<math>4b - c = -1</math>
 
<math>4b - c = -1</math>
 
 
Multiplying the second equation by <math>8</math>, we have  
 
Multiplying the second equation by <math>8</math>, we have  
 
 
<math>32b - 8c = -8</math>
 
<math>32b - 8c = -8</math>
 
 
Adding up the two equations yields  
 
Adding up the two equations yields  
 
 
<math>25b = -5</math>, so <math>b = -\frac{1}{5}</math>
 
<math>25b = -5</math>, so <math>b = -\frac{1}{5}</math>
 
 
We obtain <math>c = \frac{1}{5}</math> after plugging in the value for <math>b</math>.
 
We obtain <math>c = \frac{1}{5}</math> after plugging in the value for <math>b</math>.
 
 
Therefore, <math>a^2-b^2+c^2 = 1-\frac{1}{25}+\frac{1}{25}=\boxed{1}</math> which corresponds to <math>\text{(B)}</math>.
 
Therefore, <math>a^2-b^2+c^2 = 1-\frac{1}{25}+\frac{1}{25}=\boxed{1}</math> which corresponds to <math>\text{(B)}</math>.
 
+
This time-saving trick works only because we know that for any value of <math>a</math>, <math>a^2-b^2+c^2</math> will always be constant (it's a contest), so any value of <math>a</math> will work. This is also called [[without loss of generality]] or WLOG.
This time-saving trick works only because we know that for any value of <math>a</math>, <math>a^2-b^2+c^2</math> will always be constant (it's a contest), so any value of <math>a</math> will work.
 
  
 
==See Also==
 
==See Also==

Revision as of 20:59, 10 August 2022

Problem

Let a, b, and c be real numbers such that $a-7b+8c=4$ and $8a+4b-c=7$. Then $a^2-b^2+c^2$ is

$\mathrm{(A)\ }0\qquad\mathrm{(B)\ }1\qquad\mathrm{(C)\ }4\qquad\mathrm{(D)\ }7\qquad\mathrm{(E)\ }8$

Solution

Solution 1

Rearranging, we get $a+8c=7b+4$ and $8a-c=7-4b$

Squaring both, $a^2+16ac+64c^2=49b^2+56b+16$ and $64a^2-16ac+c^2=16b^2-56b+49$ are obtained.

Adding the two equations and dividing by $65$ gives $a^2+c^2=b^2+1$, so $a^2-b^2+c^2=\boxed{(\text{B})1}$.

Solution 2

The easiest way is to assume a value for $a$ and then solve the system of equations. For $a = 1$, we get the equations $-7b + 8c = 3$ and $4b - c = -1$ Multiplying the second equation by $8$, we have $32b - 8c = -8$ Adding up the two equations yields $25b = -5$, so $b = -\frac{1}{5}$ We obtain $c = \frac{1}{5}$ after plugging in the value for $b$. Therefore, $a^2-b^2+c^2 = 1-\frac{1}{25}+\frac{1}{25}=\boxed{1}$ which corresponds to $\text{(B)}$. This time-saving trick works only because we know that for any value of $a$, $a^2-b^2+c^2$ will always be constant (it's a contest), so any value of $a$ will work. This is also called without loss of generality or WLOG.

See Also

2002 AMC 10B (ProblemsAnswer KeyResources)
Preceded by
Problem 19
Followed by
Problem 21
1 2 3 4 5 6 7 8 9 10 11 12 13 14 15 16 17 18 19 20 21 22 23 24 25
All AMC 10 Problems and Solutions

The problems on this page are copyrighted by the Mathematical Association of America's American Mathematics Competitions. AMC logo.png